Download as pdf or txt
Download as pdf or txt
You are on page 1of 40

.

VISION IAS
www.visionias.in

TEST BOOKLET

CSAT APTITUDE TEST– (4021) – 2023


C
Time Allowed: Two Hours Maximum Marks: 200

INSTRUCTIONS

1. IMMEDIATELY AFTER THE COMMENCEMENT OF THE EXAMINATION, YOU SHOULD CHECK THAT THIS BOOKLET
DOES NOT HAVE ANY UNPRINTED OR TORN OR MISSING PAGES OR ITEMS ETC. IF SO, GET IT REPLACED BY A
COMPLETE TEST BOOKLET.

2. ENCODE CLEARLY THE TEST BOOKLET SERIES A, B, C OR D AS THE CASE MAY BE IN THE APPROPRIATE PLACE IN
THE ANSWER SHEET.

3. You have to enter your Roll Number on the Test Booklet in


the Box provided alongside. DO NOT write anything else on
the Test Booklet.

4. This Test Booklet contains 80 items (Questions). Each item is printed in English. Each item comprises four
responses (answers). You will select the response which you want to mark on the Answer Sheet. In case you feel
that there is more than one correct response, mark the response which you consider most appropriate. In any
case, choose ONLY ONE response for each item.

5. You have to mark all your responses ONLY on the separate Answer Sheet provided. See direction in the answers
sheet.

6. All items carry equal marks. Attempt all items. Your total marks will depend only on the number of correct
responses marked by you in the answer sheet. For every incorrect response one-third of the allotted Marks will
be deducted.

7. Before you proceed to mark in the Answer sheet the response to various items in the Test booklet, you have to
fill in some particulars in the answer sheets as per the instruction sent to you with your Admission Certificate.

8. After you have completed filling in all responses on the answer sheet and the examination has concluded, you
should hand over to Invigilator only the answer sheet. You are permitted to take away with you the Test
Booklet.

9. Sheets for rough work are appended in the Test Booklet at the end.

DO NOT OPEN THIS BOOKLET UNTIL YOU ARE ASKED TO DO SO


1
.
1. First 50 consecutive odd numbers are written 3. Ironite is a mixture of iron and zinc only. It is
on a black board, 1, 3, 5, 7, …. in that order. extracted from an ore that contains iron and
Two persons Anurag and Bhawna are playing a zinc in a ratio of 3:2 by weight. In the process
game of putting ‘+’ and ‘–’ sign one by one of extraction, the entire iron is retained,
between any two consecutive integers out of
but some amount of zinc is lost. If ironite
these 50 odd numbers. Both Anurag and
contains 10% zinc by weight, find the amount
Bhawna are free to put any sign (+ or –)
of ore needed to produce 5 tonnes of Ironite.
anywhere, provided there is no sign already
(a) 8.5 tonnes
placed between the two odd numbers. At the
end, when all the signs have been put between (b) 6 tonnes

all such two consecutive odd numbers, result is (c) 7.5 tonnes
to be calculated. If the result is even, Anurag (d) 7 tonnes
wins and if the result is odd, Bhawna wins.
Who will win?
4. If 17 < a < 67, how many positive integral pairs
(a) Anurag
of (a, b) are possible satisfying the equation
(b) Bhawna
(3/a) + (2/b) = 1/18?
(c) It’s a tie
(a) 7
(d) Cannot be determined.
(b) 8

(c) 9
2. Two statements S1 and S2 are given below
followed by a question. (d) 10

S1: Unit digit of 9x (i.e. 9 multiplied by x) is 1.


S2: x is a natural number. 5. When a ball is dropped, it bounces back to 3/4th
Question: What is the remainder when x is of the height from which it was dropped from .
divided by 4? If the ball is dropped from a height of 30 m,
(a) S1 alone is sufficient to answer the find the total vertical distance travelled by the
Question. ball before it comes to rest. (Neglect air
(b) S2 alone is sufficient to answer the
resistance.)
Question.
(a) 210 m
(c) Either S1 alone or S2 alone is sufficient to
(b) 240 m
answer the question.
(d) S1 and S2 together are not sufficient to (c) 180 m

answer the question. (d) 200 m

2 www.visionias.in ©Vision IAS


.
6. A wooden box of thickness 0.5 cm, length 21 Directions for the following 6 (six) items:

cm, width 11 cm and height 6 cm is painted on Read the following five passages and answer the items
that follow each passage. Your answers to these items
the inside. The box is open on the top, i.e. there
should be based on the passages only.
is no top surface/lid. The expense of painting is Passage – 1
Rs. 70. What is the rate of painting per square For most people, the climate crisis is no longer a future
scenario. Floods, heat waves and wildfires have gripped
centimeter?
our entire planet and will become stronger in years to
(a) Rs. 0.7 come. Countries in the Global South are particularly
(b) Rs. 0.5 hard hit, even though their citizens have been
emitting the least CO2. Drastic reduction of carbon
(c) Rs. 0.1
emissions is only one pillar in the fight against climate
(d) Rs. 0.2 change. Many of its consequences can no longer be
stopped. There is an urgent need for adaptation
measures so people both in cities and in rural areas have
7. Two statements are given below followed by a
the chance to cope with the effects of climate change.
question. That's why several years ago rich nations committed to

S1: Age of A is a perfect square, and age of B supporting low-income countries with $100 billion (€95
billion) annually from 2020 to help them cut emissions
is a perfect cube.
and adapt to climate change. It was a central promise
S2: A and B are twins. for more climate justice that has not been kept so far.

Question: The ages of A and B are integers 8. Based on the above passage, the following
assumptions have been made:
greater than 20 but less than 70. What are their
1. Through proper adaptation mechanisms
individual ages? and funding climate change can be arrested.

(a) S1 alone is sufficient to answer the 2. Countries from the global south are not
responsible for climate change.
Question.
3. UN could help make the rich countries
(b) S2 alone is sufficient to answer the accountable for funding the sum of $100
Question. billion for climate change.
Which of the above assumptions is/are valid?
(c) Either S1 alone or S2 alone is sufficient to
(a) 1 and 2 only
answer the question.
(b) 2 and 3 only
(d) S1 and S2 together are sufficient to answer (c) None of these

the question. (d) 1, 2 and 3

3 www.visionias.in ©Vision IAS


UPSCMATERIAL.ONLINE

.
9. What is the crucial message conveyed by the 10. Based on the above passage, the following
above passage? assumptions have been made:
1. Globalization has increased incomes
(a) Controlling carbon emissions should be the
resulting in a change in the social life of the
prime focus in the fight against climate people.
change. 2. The Indian middle class has obtained the
most benefits out of globalization.
(b) The rich countries must be made
3. The impact of globalization on women is
accountable to low-income countries facing not clear.
climate change. Which of the above assumptions is/are valid?
(c) Climate change not only affects the (a) 1 only
(b) 3 only
environment, but also threatens the very
(c) 1 and 2 only
existence of human life.
(d) All 1, 2 and 3
(d) Climate change cannot be reversed; the Passage – 3
only solution is adaptation mechanisms. The digital divide is associated with negative outcomes.
Passage – 2 It is suggested that the digital divide may cause
technological, immaterial, social, and educational types
The globalization of the Indian economy has
of inequality. Individuals who lack access to
dramatically influenced social life in India. The information technology are disadvantaged in
expansion of the middle class is said to have occurred knowledge-based societies and economies. Further, e-
government is an important means to break barriers and
as a consequence of this process. Based on
bring government assistance to all citizens. Digital
ethnographic research among lower-middle-class media usage is a strong predictor of political
families in India, there seems to be an apparent paradox participation which implies that a digital divide may
result in political alienation of those who lack digital
between women's positive perceptions of empowerment
media access.
and the overall negative impact of structural adjustment
11. Based on the above passage, the following
policies on women. Many scholars argue that assumptions have been made:
globalization has been detrimental to women due to 1. Building cyber cafes could help reduce the
digital divide.
growing structural gender inequalities, but many
2. Bridging the digital divide could help
respondents identify greater opportunities to challenge
empower citizens technologically as well as
preexisting patriarchal norms through the role models politically.
available in the globalized media. While there are Which of the above assumptions is/are valid?
(a) 1 only
increasing inequalities for households, women do not
(b) 2 only
consider these to be gender disadvantages, emphasizing
(c) Both 1 and 2
instead the opportunities for greater independence. (d) Neither 1 nor 2

4 www.visionias.in ©Vision IAS


UPSCMATERIAL.ONLINE

.
Passage – 4 13. Based on the above passage, the following
Invasive species may be one of the worst environmental assumptions have been made:
problems facing the conservation of natural areas,
1. Technology directly impacts the EQ of
because of their role in changing ecosystem function.
children.
At the same time, invasive species cause much human
suffering and economic loss. The approach to 2. Human relations have a positive impact on
eliminating invasive species can be improved by a EQ as compared to technology.
better understanding of the various types of invasive Which of the above assumptions is/are valid?
species, and the scientific hypotheses surrounding their
(a) 1 only
ability to invade novel environments. Despite the
billions of dollars spent each year, invasive species are (b) 2 only

difficult if not impossible to eliminate after they have (c) 1 and 2


been established. (d) None
12. Based on the above passage, the following
assumptions have been made:
14. D is a positive integer, which is a product of
1. Invasive species, if not eliminated in future,
could threaten the food security of people. two prime numbers. Which of the following
2. If invasive species are not allowed to cannot be a value of D?
change the ecosystem function, natural (a) 46
areas can definitely be conserved.
(b) 91
Which of the above assumptions is/are valid?
(c) 143
(a) 1 only
(d) 153
(b) 2 only
(c) Both 1 and 2
(d) Neither 1 nor 2 15. Ajay and Vijay are running towards each other
Passage – 5 at the same speed from points A and B
With an inter-connected world, we might assume that respectively. After running for some time, Ajay
today’s children have higher EQ as compared to their
sprains his ankle at point C and walks at half
predecessors. But the truth might be different. With the
his earlier speed. They finally meet at a point
boom in technology, many children are glued to their
screens and don’t get to meet other children face-to- D, such that AD = (4/10) AB. Find the ratio of
face. Relating to others and relationship management is AC/AD.
an important element that nurtures a high EQ in an (a) 1:2
individual. But technology and its addiction among
(b) 2:3
children are a hindrance in attaining high EQ levels and
this leads to many of them not being able to manage (c) 2:5

their emotions well. (d) 3:4

5 www.visionias.in ©Vision IAS


UPSCMATERIAL.ONLINE

.
Directions for the following three items: 16. The per tonne transportation cost of oil via rail
Study the pie charts given below and answer the three happens to be roughly
questions that follow. (a) Rs. 3
(b) Rs. 3.33
Chart 1 shows the percentage distribution of 12 million
(c) Rs. 4.5
tonnes of crude oil transported through different modes
(d) Rs. 2.67
over a specific period of time.

Volume Transported 17. As per the charts, it appears that the cheapest
mode of transportation is
(a) Road
Rail, 9%
(b) Rail
(c) Pipeline
(d) Ship
Road, 22%
Pipeline, 18. If the per tonne transportation costs by ship, air
49% and road are represented by P, Q and R
respectively, then which of the following is
true?
Airfreight,
11% (a) R > Q > P
(b) P > R > Q
Ship, 9% (c) P > Q > R
(d) R > P > Q
Pipeline ship

19. Ram and Shyam, who are standing 100 m away


Chart 2 shows the percentage distribution of the cost of from each other, are throwing a Frisbee towards
transporting this crude oil. The total cost was Rs. 30 each other in a straight trajectory. Initially Ram
million. has the Frisbee and as soon as he throws it
towards Shyam, they both start moving towards
Cost of Transportation each other. Shyam catches the Frisbee and
throws it back to Ram and they keep throwing
Road, 6% Rail, 12% it until they meet each other. They both throw
the frisbee at a speed of 20 m/s. If the frisbee
Airfreight,
7% travelled a total distance of 12 km and the
speeds of Ram and Shyam are in the ratio of
2:3 respectively, then find the speed of Shyam
(in metres per min). (Assume that the time
Ship, 10%
Pipeline, taken by Ram and Shyam to catch the frisbee
65% and to throw it back again is negligible, and the
frisbee travels at a uniform speed in the air).
(a) 8
(b) 6
Pipeline Ship Airfreight Road Rail
(c) 9
(d) 10

6 www.visionias.in ©Vision IAS


UPSCMATERIAL.ONLINE

.
20. Let ‘a’ and ‘b’ be two 2-digit numbers, such Passage – 2
that b is obtained by reversing the digits of a. It One of the biggest problems that solar energy
is also known that they satisfy the relation a2 – technology poses is that energy is only generated while
b2 = k2, where k is some positive integer. What the sun is shining. That means night time and overcast
is the value of a + b + k? days can interrupt the supply. The shortage created by
(a) 151 this interruption would not be a problem if there were
(b) 115 low-cost ways of storing energy as extremely sunny
(c) 154 periods can generate excess capacity. As the global
(d) 153 capacity for solar power continues to rise, nations like
Japan and other global leaders in solar energy
Directions for the following 3 (three) items: technology are focusing on developing adequate energy
Read the following three passages and answer the items storage to deal with this issue.
that follow each passage. Your answers to these items 22. Which of the following is/are the most rational
should be based on the passages only. and logical inference/inferences that can be
Passage – 1 drawn from the passage?
Ecosystem services, defined as the benefits to human 1. Tropical countries with year-long
welfare provided by organisms interacting in availability of sunshine are less dependent
ecosystems, are considered to be at risk. Pollination by on the storage of solar energy.
wild animals is a key ecosystem service. Crop 2. In future, the cost of storage of solar energy
pollination is commonly cited as an example of an may reduce due to the involvement of
endangered ecosystem service as detailed studies of the global leaders.
crop pollination systems are incomplete or out of date. Select the correct answer from the code given
Animal pollination is important to the sexual below:
reproduction of many crops and the majority of wild (a) 1 only
plants, which can also be important for providing (b) 2 only
calories and micronutrients for humans. Furthermore, (c) 1 and 2
the decline of pollinating species can lead to a parallel (d) None
decline of plant species. Passage – 3
21. Which of the following is/are the most rational New advancements in technologies ranging from
and logical Inference/Inferences that can be robotics and drones to computer vision software have
made from the passage? completely transformed modern agriculture. The
1. To protect this ecosystem service, holistic primary goal of farm automation technology is to cover
contemporary research in crop pollination easier, mundane tasks. Some major technologies that
may help. are most commonly being utilized by farms include
2. Loss of pollinator species can have grave harvest automation, autonomous tractors, seeding and
consequences for human nutrition. weeding, and drones. Farm automation technology
Select the correct answer from the code given addresses major issues like a rising global population,
below: farm labour shortages, and changing consumer
(a) 1 only preferences. The benefits of automating traditional
(b) 2 only farming processes are monumental by tackling issues
(c) Both 1 and 2 from consumer preferences, labour shortages, and the
(d) Neither 1 nor 2 environmental footprint of farming.

7 www.visionias.in ©Vision IAS


UPSCMATERIAL.ONLINE

.
23. Which one of the following statements best 27. There are 25 students in a class, and the number
reflects the central idea of the passage? of students who scored 5, 7 and 3 marks in a
(a) The supply chain of agricultural produce
test are 8, 10 and 7 respectively. The teacher
will transform with evolving technologies.
(b) Agricultural technology has the potential to can decide how many students should go to the
solve socio-economic issues along with the remedial class, but can't pick specific students
environment. for it. Once the number is decided, the students
(c) There is a dire need for automation of
are selected at random. The teacher wants every
agriculture in India to help alleviate the
status of marginal farmers. student in the class to have atleast three others
(d) Application of modern technology will who scored the same marks in the examination
increase the input cost of farmers. as him/her. What is the minimum number of
students who should be sent to the remedial
24. Let x, y and z be distinct integers, wherein x
and y are odd and positive, and z is even and class for this to happen?
positive. Which one of the following statements (a) 19
cannot be true? (b) 22
(a) y(x − z)2 is even
(c) 18
(b) y2(x − z) is odd
(c) y(x − z) is odd (d) 16
(d) z(x − y)2 is even
28. If X and Y are multiples of 7 with X>Y>0,
25. Raman was asked to add first few natural
which of the following statements is not always
numbers (i.e., 1 + 2 + 3 + …) as long as he
wanted. When he stopped, he gave the sum as true?
575. When the teacher declared the result was (a) X3 – Y3 is divisible by 49.
wrong, Raman discovered that he had missed
(b) HCF of X and Y is a multiple of 7.
one number in the sequence during addition.
What was the number that he missed? (c) LCM of X and Y is a multiple of 49.
(a) Less than 10 (d) X2 – Y2 is a multiple of 49.
(b) 10
(c) 15
29. A six-digit number N is formed using the digits
(d) More than 15
0, 3, 6 and 9 only. Each of the digits is used at

26. A number N gives a remainder of 7 when least once. It was found that N is divisible by
divided by D and a remainder of 20 when 18. What is the ten’s digit of the smallest
divided by 2D. What will be the remainder possible such six-digit number?
obtained when 2N is divided by D?
(a) 0
(a) 0
(b) 1 (b) 9
(c) 6 (c) 6
(d) 3 (d) 3

8 www.visionias.in ©Vision IAS


UPSCMATERIAL.ONLINE

.
30. If the numerator of a fraction is increased by 31. Consider the following:
a% and denominator of the fraction is increased 1. Pollution
2. Non-discriminatory education
by b%, then what will be the ratio of original
3. Irrigation
fraction to new fraction?
4. Quality healthcare
(a) a/b 5. Human Survival
(b) (a +1)/(b + 1) 6. Avariciousness
(c) (100 + b)/(100 + a) Which of the above issues have been
highlighted by the passage?
(d) b/a
(a) 1, 2, 4 and 5 only
(b) 1, 2, 4, 5 and 6 only
Directions for the following 3 (three) items: (c) 3, 4, 5 and 6 only
Read the following three passages and answer the items (d) 2, 4 and 5 only

that follow each passage. Your answers to these items Passage – 2


In cities, in high-rises, not many of us know our
should be based on the passages only.
neighbors anyway. Parents might not like cricket.
Passage – 1 Friends might not share our interest in sports. The
Universal and affordable access to quality education intrusion of others is probably not welcomed by all.
Especially, for this generation which has grown up in
and healthcare must be the focus of our attention. Rural
bubbles for most things — entertainment, online
India cannot be left behind in this mission. Improving studies, social media — even if, paradoxically, the
rural infrastructure across the country must be fast- surfeit of lonely pursuits has reduced the quality of
tracked and integrated into the larger development solitude.
32. Which one of the following statements best
narrative. Promotion of the mother tongue will
reflects the suggestion conveyed by the
revolutionize the educational landscape by making it passage given above?
more inclusive and equitable. (a) People should avoid living in cities.
With his unbounded greed, man has caused irreversible (b) In cities, people should become more
socialized.
damage to the environment. Today, climate change is a
(c) In search of loneliness, we have shortened
global phenomenon. Conservation is our only hope for the quality of solitude.
the survival of the planet and we must direct our (d) Cities have become a breeding ground for
collective energies toward this end with missionary loneliness.
Passage—3
zeal. Our scriptures mirror the cosmic vision of our
Earlier, the rural always existed in Hindi films. It was a
ancient culture. They are replete with examples of the regular narrative trope: characters from rural areas
worship of the divine in the elements — rivers, would always be arriving in the city. Those films
mountains, holy plants and trees. We only have to go created a split between the rural and the urban: the rural
was seen as pure and the urban as impure, the rural as
back to our moorings to draw inspiration and strive to
innocent and the urban as corrupt. They created a
secure the future of succeeding generations. binary between the village and the city.

9 www.visionias.in ©Vision IAS


UPSCMATERIAL.ONLINE

.
33. According to the passage, which of the Directions for the following 2 (two) items:
following is correct about Hindi films? Study the graph given below carefully and answer the
(a) Now, Hindi films portray only ‘the urban’.
questions that follow.
(b) It is solely because of Hindi films that there
exists a split between ‘the rural’ and ‘the The bar graph given below shows the data of the
urban’. production of paper (in lakh tonnes) by three different
(c) Earlier, Hindi films portrayed the rural as
companies X, Y and Z.
‘good’ and the urban as ‘bad’.
(d) Both (b) and (c) Production of Paper (in lakh tonnes) by
Three Companies X, Y and Z
34. The sum of the areas of two circles, which 55
45 45
50 50
50
touch each other externally is 153π. If the sum 45 40 40 40
40 35 35 35 35 35
35 30
of their radii is 15, find the ratio of the larger to 30 25 25
25
the smaller radius. 20
15
(a) 4 10
5
(b) 2 0
2006 2007 2008 2009 2010
(c) 3
x Y Z
(d) None of these

36. For which of the following years, is the


35. Two statements are given below followed by a
question. percentage rise/fall in production from the

S1: The cost of three caramel pops, five toffee previous year the maximum for Company Y?
bites and seven mango pops is Rs. 68. (a) 2007
S2: The cost of two caramel pops, three toffee
(b) 2008
bites and four mango pops is Rs. 41.
Question: Priya went to a candy shop to buy (c) 2009
some caramel pops, toffee bites and mango (d) 2010
pops. What is the total price of one caramel
pop, one mango pop and one toffee bite?
Which one of the following is correct in respect 37. What is the ratio of the average production of
of the Statements and the Question? Company X in the period 2008-2010 to the
(a) S1 alone is sufficient to answer the average production of Company Y in the same
Question.
period?
(b) S2 alone is sufficient to answer the
Question. (a) 1:1
(c) S1 and S2 together are sufficient to answer (b) 15:17
the question.
(c) 23:25
(d) S1 and S2 together are not sufficient to
answer the Question. (d) 27:29

10 www.visionias.in ©Vision IAS


UPSCMATERIAL.ONLINE

.
38. If value of a is between – 6 and – 3 and value Directions for the following 3 (three) items:
of b is between – 3 and 3, then the value of Read the following two passages and answer the items
(a2 – b2) must be in between which of the that follow each passage. Your answers to these items

following ranges? should be based on the passages only.


Passage – 1
(a) – 36 and 9
Scientists might have a solution to growing concerns of
(b) – 36 and – 9
climate change hampering crop productivity.
(c) 0 and 9
Researchers have identified genes in Africa’s local food
(d) 0 and 36
crops that can improve resistance to drought.
Incorporating these genes through hybridisation might
39. Ramesh bought some articles at a price which improve crop yield that is affected by increasing global
was 10% lower than its marked price. He then temperatures and heatwaves. Countries like India and
sold 50% of the articles at a price 10% above Pakistan are also likely to be affected by the prolonged

the marked price. After this he sold the next heatwaves, the United Nations Environment
Programme (UNEP) estimated. The UN body is
25% articles at a price which was 20% lower
responsible for coordinating responses to environmental
than the marked price. If Ramesh wants to earn
issues. Orphan crops are nutritious local food crops that
a profit of 10% in the entire transaction, then at
could play a crucial role in combating hunger. These
what price should he sell the remaining 25%
crops are not traded internationally but have adapted
articles? themselves to grow in harsh weather conditions.
(a) 4% above the marked price Africa’s arid environment is not suited for crops like
(b) 4% below the marked price rice and maize. Orphan crops like finger millets, little
(c) 6% above the marked price millet, African yam bean, jojoba and jatropha are

(d) 6% below the marked price commonly found on the continent.


41. Based on the above passage, the following
assumptions have been made:
40. A shopkeeper sells 1 kg rice to two customers
1. Africa has the potential to fight hunger
Rani and Vani. For Rani he charges exactly the
without dependence on other countries.
cost price but under weighs the quantity by
2. UNEP should promote the trade of orphan
10%. For Vani he sells at 20% more than the crops from Africa to other hunger-affected
cost price but over weighs the quantity by 10%. continents.
What is his overall profit/ loss percentage? Which of the above assumptions is/are valid?
(a) 10% loss (a) 1 only
(b) 10% profit (b) 2 only
(c) 9.1% loss (c) Both 1 and 2

(d) 9.1% profit (d) Neither 1 nor 2

11 www.visionias.in ©Vision IAS


UPSCMATERIAL.ONLINE

.
Passage – 2 44. There is a circle of radius 5 cm that
India would reduce fossil fuels, generate more circumscribes a quadrilateral. It is further
renewable energy, reduce carbon emissions and
known that 2 of the adjacent sides of the
intensity, and eventually become a net-zero emitter by
2070. The goals are ambitious. They are built around quadrilateral are 6 cm and 8 cm. Determine the
the levers which, at the policy level, are most likely to maximum possible area of the quadrilateral.
create a positive impact on climate. Can policy
initiatives alone fulfil India’s commitments toward (a) 38 cm2
arresting climate change? Actions that are driven top- (b) 52 cm2
down, via government policy, no doubt set the stage for
(c) 64 cm2
change. But unless people shift their attitude and
behaviour in tune with the policy initiatives, change (d) 49 cm2
will not be able to retain its momentum.
42. Based on the above passage, the following
45. In a polygon, all the interior angles are either
suggestions may be made:
1. India, with the help of renewable energy, 90 degrees or 270 degrees. If the number of
would be able to control carbon emissions interior angles which are 90 degrees is 31, find
completely by 2070.
the number of angles which are 270 degrees?
2. Government may involve citizens in
policymaking and execution for achieving (a) 31
the target of net-zero emission.
(b) 35
Which of the above suggestions is/are valid?
(c) 27
(a) 1 only
(b) 2 only (d) Cannot be determined
(c) Both 1 and 2
(d) Neither 1 nor 2
46. In a school, 90% of the students faced problems

43. What is the crucial message conveyed by the in Mathematics, 80% of the students faced
above passage? problems in Computers, 75% of the students
(a) Climate change cannot be reversed only
faced problems in Sciences, and 70% of the
with the commitments of India.
(b) Climate change is a global issue, and students faced problems in Social Sciences.
people across countries must change their Find the minimum possible percent of the
attitude towards it.
students who faced problems in all the four
(c) Indian government should consult
specialised global agencies in the subjects.
formulation of climate change policy. (a) 15%
(d) Policy framework along with targets should
(b) 25%
prescribe citizens' behaviour for
contributing to meeting net-zero emission (c) 26%
targets by 2070.
(d) 32%

12 www.visionias.in ©Vision IAS


UPSCMATERIAL.ONLINE

.
47. If two letters are taken at random from the 50. P is a prime number > 71. If N = (P + 2)
word HOME, what is the probability that none (P − 1)(P + 1)(P − 2), then which of the
of the letters would be vowels? following will definitely divide N for all the
(a) 1/6 values of P?
(b) 1/2 (1) 120
(c) 1/3
(2) 240
(d) 1/4
(3) 360
(4) 480
Direction for the following 2 (two) items.
(a) 1 and 2 only
Study the table given below carefully and answer the
questions that follow. (b) 1 and 3 only
The following table gives the percentage distribution of (c) 2 and 4 only
population of five states, P, Q, R, S and T on the basis (d) 1, 2 and 3 only
of poverty line and also on the basis of gender.
State Percentage of Proportion of Males and Females Directions for the following 3 (three) items:
Population Below poverty Above
Read the following two passages and answer the items
below the line Poverty Line
Poverty Line that follow each passage. Your answers to these items
M F M F
should be based on the passages only.
P 35 5 6 6 7
Q 25 3 5 4 5 Passage – 1
R 24 1 2 2 3 India which will soon have twice the number of Internet
S 19 3 2 4 3 users as the U.S. is a large market for all kinds of new
T 15 5 3 3 2 technologies. While this presents a huge opportunity,
48. If the male population above poverty line for the domestic industry has not yet managed to derive the
State R is 1.9 million, then the total population benefits. For instance, the country is operating far
of State R is below its potential in electronic manufacturing.
(a) 4.5 million Electronic goods and components are the second largest
(b) 4.85 million item, after oil, in India’s import bill. Also, the country’s
(c) 5.35 million imports are almost five times its exports in this industry
(d) 6.25 million (based on 2020-21 data). High-value electronic
components needed in the manufacture of, say, mobile
49. What will be the number of females above the phones are technology and design-intensive. Big
poverty line in the State S if it is known that the multinational companies control these technologies and
population of State S is 7 million?
corner the bulk of the revenues. China has used its large
(a) 3 million
market size as a bargaining chip in negotiations with
(b) 2.43 million
foreign firms: stay in our markets only if you localize
(c) 1.33 million
production and share technologies with the local firms.
(d) 5.7 million

13 www.visionias.in ©Vision IAS


UPSCMATERIAL.ONLINE

.
51. Based on the above passage, the following Passage – 2

assumptions and suggestions have been made: While governance is often equated with government, it
involves much more. Governance occurs at all levels
1. Indians have a high rate of consumption of
and encompasses the ways that formal government,
electronics.
non-governmental groups, community organizations
2. India may use its large market size as a and the private sector manage resources and affairs.
bargaining chip for the promotion of its Three factors that largely determine the efficacy of any

electronics industry. system of governance are the quality of leadership, the


characteristics of the governed, and the nature of the
Which of the above assumptions and
structures and processes employed to exercise authority
suggestions is/are valid?
and meet human needs. The capacity of any institution
(a) 1 only to effect and manage change, and to respond creatively
(b) 2 only to challenges that lie before it, increases with the
development of several critical skills. These include the
(c) 1 and 2
ability to maintain a clear perception of social reality
(d) None
and the forces operating in it; to properly assess the
resources of the community; and to implement
52. Which one of the following statements best decisions with an openness and flexibility that avoid all

reflects the crux of the passage? traces of dictatorial behaviour, among others. This
constellation of skills must draw on both intellectual
(a) Union government should trade with the
and moral resources.
USA not only in products but also in
53. Based on the above passage, the following
services like technology. assumptions have been made:
(b) Domestic companies should invest in 1. Not only the government but citizens are
human resources to excel in the also responsible for governance outcomes.
2. Rigid and closed implementation of
technologies of the electronic industry.
decisions reduces the capacity of governing
(c) Despite a large user base, India is not able
institutions.
to scale electronics production due to
Which of the above assumptions is/are valid?
technological limitations. (a) 1 only
(d) Big multinational companies should shift (b) 2 only
their manufacturing base from China to (c) 1 and 2

India. (d) None

14 www.visionias.in ©Vision IAS


UPSCMATERIAL.ONLINE

.
54. A super lotto has all the two-digit numbers 57. From the first 25 natural numbers, how many
written on various balls, which have been kept arithmetic progressions (APs) of 6 terms can be
in a bag. Number on each ball is different. If formed such that the common difference of the
two balls are chosen at random without AP is a factor of the 6th term?
replacement, then what is the probability that (a) 31
their numbers have the same units digit and the (b) 32
product of these numbers also has the same (c) 30
units digit? (d) 28
(a) 2/5
(b) 2/25 58. The ratio of incomes of Anil and Ajit is 5:7.
(c) 1/25 The ratio of their expenditures is 3:4. Which of
(d) None of the above the following is a possible ratio of their
savings? (Savings = Income - Expenditure)
55. There are 100 people in a group. Each person, (a) 4:5
while working individually, can complete a (b) 3:4
piece of work in 100 days. One person starts (c) 1:1
working on the project. On the second day, (d) 12:17
another person joins him, on the third day, one
more person joins the first two and this process
59. 29 paintings are displayed in an art exhibition,
goes on till the work is completed. On which
which are made by using either oil colours or
day will the work get completed?
water colours or both. Oil and water colours
(a) 13th day have been used in making 16 and 19 of these
(b) 14th day paintings respectively. What is the number of
(c) 12th day paintings in which both types of colours have
th
(d) 15 day been used?
(a) 5
56. If x + y = 36, then which of the following must (b) 6
be true? (c) 3
1. Both x and y must be positive. (d) 8
2. If x is positive, y must be negative.
3. If x is negative, y must be positive. 60. How many pairs of integers exist such that the
Select the correct answer using the code given difference between their product and sum is
below. 72?
(a) 1 only (a) 6
(b) 3 only (b) 4
(c) 2 only (c) 2
(d) Neither 1 nor 2 nor 3 (d) 13

15 www.visionias.in ©Vision IAS


UPSCMATERIAL.ONLINE

.
Directions for the following 3 (three) items: 62. Which one of the following statements best
Read the following three passages and answer the items
reflects the crux of the passage?
that follow each passage. Your answers to these items
should be based on the passages only. (a) Incineration is not the best technological
Passage – 1 solution for waste processing in India.
The ignorance of class differences that are necessitated
in democracy’s pursuit of equality makes it vulnerable (b) Reducing consumption will solve the issue
to class conflict. By institutionalizing class disparities of waste generation.
into a balanced representative structure, republicanism
(c) Incinerators shall be established in and
attempts to preserve order between distinctive
subdivisions of society. Also, the pursuit of complete around the landfills for faster processing of
‘negative liberty’ in pure democracies is susceptible to
waste.
anarchic disorder, as it disregards the law. The
responsibility to uphold the law, typically maintained in (d) India should develop customized
the form of a constitution, by both the government and
incinerators to ensure the processing of
the individual is essential in a republic for the
preservation of societal order. organic waste.
61. Which one of the following statements best
Passage – 3
reflects the crux of the passage?
(a) Limitations of democracy regarding social Especially in the last century, most Western countries
order can be handled through the have experienced significant demographic changes with
constitution.
(b) Democracy due to inherent flaws is not the a continuing increase in the number of older people
best political model for governance. who face medical and functional challenges, as well as
(c) Only governments following a constitution
can guarantee the preservation of social diseases that are age-specific but have often originated
order. in people’s younger years. Most of these diseases
(d) The pursuit of equality in a democracy is
including obesity, cardiovascular heart diseases (CHD)
the bedrock for preventing class conflicts.
Passage – 2 or type 2 diabetes mellitus are caused by civilisation.
Hyperconsumption is a curse of our modern times. Most NCDs primarily result from unhealthy lifestyles
Humans generate monumental amounts of waste, a
sizeable portion of which is disposed of in landfills and including the consumption of too much or unhealthy
through waste-to-energy incinerators. However, billions food, too much alcohol and excessive smoking habits,
of tonnes of garbage, including microplastics, never
make it to landfills or incinerators and end up in the combined with physical inactivity. More specifically,
oceans. This garbage chokes marine life and disturbs inactivity and unhealthy eating habits are associated
zooplankton, which is vital to the elimination of carbon
with weight gain. Overweight and obesity are the major
dioxide from the atmosphere. In India, nearly 60% of
the household waste is wet organic waste, with low underlying causes of modern diseases such as CHD or
calorific value. This makes options such as waste-to-
type 2 diabetes mellitus.
energy incinerators inefficient.

16 www.visionias.in ©Vision IAS


UPSCMATERIAL.ONLINE

.
63. Which one of the following statements best 66. Two statements S1 and S2 are given below
reflects the practical implication of the with regard to two numbers followed by
passage? question.
(a) Prevention of NCDs can be done through S1: a > b3
physical activity and making healthy food S2: a > b6
choices. Question: If the product of a and b is a positive
(b) Indians should learn from western countries number, is (a + b) > 0?
about the importance of physical activity
Which one of the following is correct in respect
for good health.
of the Statements and the Question?
(c) Due to physical inactivity, any kind of non-
(a) S1 alone is sufficient to answer the
communicable disease can develop at any
Question.
age of life.
(b) S2 alone is sufficient to answer the
(d) Unhealthy food, alcohol, and smoking
Question.
causes CHD or type 2 diabetes mellitus.
(c) Either S1 alone or S2 alone is sufficient to
answer the question.
64. Two statements S1 and S2 are given below
(d) S1 and S2 together are sufficient to answer
followed by a question.
the question.
S1: Price of mixture A is Rs. 40 per litre and
price of mixture of B is Rs. 32 per litre.
67. How many distinct natural numbers 'n' are there
S2: Price of mixture C is Rs. 50 per litre.
such that, amongst all its divisors, greater than
There are three different mixtures of milk and
1 and less than 'n', the largest divisor is 21
water—mixture A, mixture B, and mixture C.
times the smallest divisor?
In what ratio should mixtures A and B be
(a) 0
mixed with mixture C to get the average price
Rs. 35 per litre? (b) 1
(a) S1 alone is sufficient to answer the (c) 2
Question. (d) Infinite
(b) S2 alone is sufficient to answer the
Question. 68. It takes 10 men and 20 women to complete a
(c) Either S1 alone or S2 alone is sufficient to project in 48 days. After the first day, a woman
answer the question. was replaced by a man and it took them 45
(d) S1 and S2 together are NOT sufficient to more days to complete the work. If 10 men and
answer the question. 20 women had worked on the same project, and
after 2 days, if 2 of the women had been
65. If we have two different prime numbers x and replaced by 2 men, how many more days would
y, both greater than 2, then which of the it have taken to finish the project? (Round up to
following must be true? the nearest integer)
(a) x − y = 23 (a) 43
(b) x + y ≠ 87 (b) 44
(c) Both (a) and (b) (c) 40
(d) None of these (d) 42

17 www.visionias.in ©Vision IAS


UPSCMATERIAL.ONLINE

.
69. A certain number of people take 40 hours to Passage – 1
process a fixed quantity of a substance. The India’s structural transformation, particularly since

substance is priced at Rs x per unit which is 1991, has been slow and unique. Despite abundant low-
skilled labour, our growth trajectory has mostly skipped
determined by the labour costs and other
manufacturing, growing instead on the back of a far
costs. After a new technology is developed, the
smaller, high-skilled services sector. Consequently, the
same number of people take 20 hours to
bulk of jobs our economy generated even in its peak
process the same amount of the substance. If
growth years was in the largely informal, low-value add
the cost of technology is negligible, how much construction sector. The distributional consequences of
would the new price of the processed substance this have been significant. Under-employment and low
per unit quantity be? (Assume that the other inter-generational mobility have been persistent

costs remain the same.) features of the Indian economy resulting in deep
inequalities. Most of India continues to live in
(a) > x
extremely precarious economic conditions with limited
(b) < x/2
opportunity. Growth lifted a large population out of
(c) x/2
poverty. However, as the World Bank data show, most
(d) > x/2 of those (40 per cent of the population) who escaped
poverty between 2005-2012 moved into what they call
70. The cost of 4 pencils, 2 erasers and a sharpener the vulnerable group — one income shock away from

together is Rs. 80, whereas the cost of 1 pencil, falling below the poverty line.
71. Based on the above passage, the following
2 erasers and a sharpener together is Rs. 35.
assumptions have been made:
Find the combined cost of 2 pencils, 2 erasers
1. Availability of low skilled labor may
and 1 sharpener.
promote the development of manufacturing
(a) Rs. 60
sector.
(b) Rs. 50 2. Service sector can thrive even on a small
(c) Rs. 40 workforce.

(d) Cannot be determined 3. The peak growth year of 2005-2012 helped


the service sector grow faster.
Which of the above assumptions is/are valid?
Directions for the following 4 (four) items:
(a) 1 and 2 only
Read the following three passages and answer the items
(b) 2 and 3 only
that follow each passage. Your answers to these items
(c) 1 and 3 only
should be based on the passages only. (d) 1, 2 and 3

18 www.visionias.in ©Vision IAS


UPSCMATERIAL.ONLINE

.
72. According to the passage given above, India’s Passage – 3
unique structural transformation has led to Education is a journey, which gives the art of living, not
which of the following? just the livelihood. It makes us learn how to nurture our
1. A category of people just outside the life and be more creative. Education makes us
bounds of poverty. understand our conflicts. Thus, education is not merely
2. Decline in the share of agriculture in the learning facts but is training our minds to think.
overall economic growth. Education systems must provide opportunities to every
Select the correct answer using the code given individual to learn through experience and should help
below. to develop critical thinking and problem-solving skills.
(a) 1 only However, in India, the education system has evolved in
(b) 2 only a completely different manner. Instead of focusing on
(c) Both 1 and 2 critical thinking, expressing new ideas and debating and
(d) Neither 1 nor 2 writing critically on any issue, our students are forced
to learn through the rote route.
Passage – 2
74. Which of the following is/are the most rational
The debate around how the matters of the ‘mind’ are
and logical Inference/Inferences that can be
associated with the male and the ‘body’ invariably
made from the passage?
linked to the woman is not new. And of course, the
mind is superior to the body. In this regard, the feminist 1. Indians cannot think critically and be
critique of French philosopher René Descartes’s creative.
Cartesian duality is an important intervention. The 2. One way to improve the outcome of
mind-body dualism has led to gender inequality by education is to include learning by
privileging the ‘rational’ mind over the ‘irrational’ or experience and reduce rote learning.
emotional body. Women’s voices have been suppressed Select the correct answer using the code given
and their silence has been treated as ‘golden’. What we below:
need to understand, and question is the politics behind (a) 1 only
recognizing the domain of intellect as essentially a male (b) 2 only
prerogative. (c) Both 1 and 2
73. Which of the following is/are the most rational (d) Neither 1 nor 2
and logical Inference/Inferences that can be
made from the passage? 75. In a class consisting of 150 students, one more
1. Women should be treated equally with men student is added and as a result the average
even in aspects of intellect. weight of class increases by 0.05 units.
2. Descartes may be said to be responsible for Thereafter, from this class one student is taken
gender inequality in society to some extent. out and the average weight of the class is
3. Gender equality cannot be achieved reduced by 0.02 units. If the sum of the weights
without social change. of the two students mentioned is 2x+b, where x
Select the correct answer from the code given is the original average weight of the class of
below: 150 students, then find the value of b.
(a) 1 and 2 only (a) 7.50
(b) 2 only (b) 3.05
(c) None of the above (c) 10.60
(d) 1, 2 and 3 (d) 11.10

19 www.visionias.in ©Vision IAS


UPSCMATERIAL.ONLINE

.
76. A, B and C are 3 varieties of beer which 78. In 1981 and 1987, 50 million and 60 million
contain starch, hops and water in varying
people were employed in manufacturing sector
proportions. A contains x% starch and y%
hops. B contains y% starch and y% water. C respectively. What was the difference between
contains x% hops and y% water. When A, B
the number of employees employed in small
and C are mixed in the ratio 2:3:5, the final
mixture has 38% starch and 28% hops. Find the establishments in 1981 and 1987?
sum of x and y.
(a) 8 million
(a) 50
(b) 26 (b) 12 million
(c) 13
(c) 6 million
(d) 65
(d) 4 million
77. A cricket team of 11 is to be selected from 7
batsmen and 6 bowlers which are available.
The team must have at least 5 bowlers. In how 79. In 1996 there were a total of 100,000
many ways can it be done?
employees in manufacturing sector and 750
(a) 21
(b) 42 medium establishments. What was the average
(c) 84 number of employed persons per medium
(d) 63
establishment unit in that year?
Directions for the following 2 (two) items: (a) 100
The following graph shows the percentage share of
(b) 75
small, medium and large establishments in total
employment provided by manufacturing sector. Study (c) 125
the graph carefully and answer the items that follow.
(d) 1000
Percentage share of small, medium
and large establishments in jobs in
manufacturing sector 80. A book contains 57 stories, each with fewer
80
than 58 pages. The first story starts on the 2nd
70
60 page and each story thereafter, starts on a fresh
50 page. What is the largest possible number of
40
stories than can begin on an odd numbered
30
20 page?
10 (a) 29
0
(b) 56
Small establishments Medium establishments (c) 55
Large establishments
(d) 28

20 www.visionias.in ©Vision IAS


UPSCMATERIAL.ONLINE

.
VISIONIAS
www.visionias.in

ANSWERS & EXPLANATION


APTITUDE TEST–Test (4021) – 2023

1.(a)
Whatever sign we put between two odd numbers, resultant of 50 odd numbers = Even
Hence, Anurag will win.
Hence, option (a) is correct.

2.(d)
Using S1 alone:
Unit digit of 9x is 1.
x can be 9 or 19 or 29, …. So, remainder obtained when x is divided by 4 can be 1 or 3.
Hence S1 alone is NOT sufficient.
Using S2 alone:
x is a natural number.
Value of x can be any natural number. As a result, remainder obtained when x is divided by 4 will NOT
have a unique value. Hence S2 alone is NOT sufficient.
Both statements together are not sufficient either.
Hence, option (d) is correct.

3.(c)
Zinc present in 5 tonnes of Ironite = 0.5 tonnes and
Iron present in 5 tonnes of Ironite = 4.5 tonnes.
Since the ratio given in ore is 3:2, the weight of iron = 3x and the weight of zinc = 2x
Some zinc is lost, but the amount of iron will remain the same in both.
Hence, 4.5 = 3x
or x = 1.5
Total weight = 3x + 2x = 5x = 5 × 1.5 = 7.5 tonnes
Thus, 7.5 tonnes of ore is needed to produce 5 tonnes of Ironite.
Hence, option (c) is correct.

4.(b)
Given equation:
(3/a) + (2/b) = 1/18
The given equation can be rewritten as 3b + 2a = (ab/18)
Or 54b + 36a = ab
Or ab - 36a - 54b = 0
In order to factorize, we add 36×54 = 1944 to both sides.
ab - 36a - 54b + 1944 = 1944
Or a(b – 36) – 54(b – 36) = 1944
Or (a - 54) (b - 36) = 1944.
This equation can be written as (a - 54) (b - 36) = 1944 = 23×35
For 17 < a < 67, the following ordered pairs satisfy the given equation.:
(55, 1980), (56, 1008), (57, 684), (58, 522), (60, 360), (62, 279), (63, 252), (66, 198), i.e., 8 pairs.
Hence, option (b) is correct.

1 www.visionias.in ©Vision IAS


UPSCMATERIAL.ONLINE

.
5.(a)
Here, when the ball is dropped, it bounces back to 3/4 of the height it was dropped from, and then falls
again. So, in other words it covers 3/4 of its original drop-height 2 times - going up and then down. The
same will happen every time.
Initial height is 30 m and in each bounce, it rebounds to 3/4th of the distance it fell.
After the first bounce, the height reached is (3/4)×(30).
After the second bounce, the height reached = (3/4)×[(3/4)×(30)] = 30×(3/4)2
After the third bounce, the height reached = (3/4)×[(3/4)×(3/4)×(30)] = 30×(3/4)3
So, Total distance travelled = 30 + 2×(30)×(3/4) + 2×30×(3/4)2 + 2×30×(3/4)3 + ……..
= 30 + 2 × [(30)×(3/4) +(30)× (3/4)2 + (30)×(3/4)3 + ……..]
The given series is in the form of a geometric progression.

So, total distance covered = Distance travelled first time + 2 × {a ÷ (1 - r)}


Here a = 30×(3/4), common ratio (r) = 3/4
So, total distance covered = 30 + 2 × [30×(3/4)/[(1 – 3/4)] = 30 + 2 × [30×(3/4)×4] = 30 + 180 = 210 m

6.(c)
Inside dimensions of cube are as follows:
Length = 21 - 2 × 0.5 = 20 cm
Width = 11 - 2 × 0.5 = 10 cm
Height = 6 – 0.5 = 5.5 cm (as top is open)
So total painted area = 2 (20 × 5.5) + 2 (10 × 5.5) + (10 × 20)
= 220 + 110 + 200
= 530 cm2
Cost of painting 530 cm2 is Rs. 70
So, cost per cm2 = 70/530 = 0.1

7.(d)
Using both the statements individually, we can't solve for the ages of A and B.
However, using both the statements together, we can conclude that A = B and the required number is both
a perfect square and a perfect cube.
So, A = B = x6 for some x.
Only 64 satisfies this condition for x = 2.
So, we can answer the question using both the Statements together, but not by using any of the two
statements individually.
Hence, option (d) is correct.

8.(c)
Assumption 1 is incorrect. The statement mentions arresting climate change with the help of funding and
adaptation mechanisms. However, the lines “Drastic reduction of carbon emissions is only one pillar in
the fight against climate change. Many of its consequences can no longer be stopped”, show that climate
change has, now, reached a point where it cannot be arrested or stopped. Therefore, it is an incorrect
assumption.
Assumption 2 is incorrect. The only line regarding countries from the global south is “Countries in the
Global South are particularly hard hit, even though their citizens have been emitting the least CO2.” It
shows that they too have been emitting carbon dioxide. “Least amount” is only a relative connotation and
can be very high in absolute terms. Therefore, we cannot assume that they are not responsible for climate
change. As per the passage, certainly, the major emissions have been from rich countries, but the
assumption mentioned completely absolves the countries from the global south, which is incorrect.
Assumption 3 is incorrect. The statement is beyond the scope of the passage because the passage does
not discuss the role of the UN in making rich countries accountable for the failed promise of funding for
addressing the impacts of climate change. Thus, it is an incorrect assumption.

2 www.visionias.in ©Vision IAS


UPSCMATERIAL.ONLINE

.
9.(d)
Option (a) is incorrect. This option is not correct because the line “Drastic reduction of carbon
emissions is only one pillar in the fight against climate change”, reflects that we have reached a point
where only controlling carbon emissions will not address the adverse impacts of climate change. Rather,
focussing on “adaptation measures” needs focus, “so people both in cities and in rural areas have the
chance to cope with the effects of climate change.”
Option (b) is incorrect. This option seems to be correct, but it is not the crucial message because the core
theme of the passage is not about making the rich countries accountable, but that climate change is
irreversible and needs the world population to urgently adapt to it.
Also, as the rich countries have not fulfilled their promise of funding $100 billion, this option could be a
rational implication but not a crucial message.
Option (c) is incorrect. The given option is very broad but beyond the scope of the passage, as it is not
inspired by the information given in the passage. The passage does not discuss climate change threatening
the very existence of human life. Therefore, this cannot be the crucial message of the passage.
Option (d) is correct. This option captures the real essence of the passage. The lines “Many of its
consequences can no longer be stopped. There is an urgent need for adaptation measures so people both
in cities and in rural areas have the chance to cope with the effects of climate change” reflect that climate
change cannot be reversed and that adaptation is a must for coping with climate change. Therefore, the
message in this option is the crucial message of the passage.

10.(b)
Assumption 1 is incorrect. The statement is beyond the scope of the passage because the passage does
not mention or discuss anything regarding income or increased incomes. So, this is not a correct
assumption. The passage instead discusses the increased opportunities for women in the globalised world.
Assumption 2 is incorrect. The line “The expansion of the middle class is said to have occurred as a
consequence of this process” reflects that there has been a considerable impact of globalization on the
Indian middle class, but does not compare it with middle classes of other countries. Also, the passage does
not compare the impact of globalization on different classes in India.
Assumption 3 is correct. This statement is correct because of the following lines from the passage,
“Many scholars argue that globalization has been detrimental to women due to growing structural gender
inequalities, but many respondents identify greater opportunities to challenge preexisting patriarchal
norms through the role models available in the globalized media”. These lines represent two contrasting
opinions about the impact of globalization on women. Scholars say that it has been detrimental, but
respondents say that it has benefitted as it provides opportunities to challenge the patriarchal norms.
Therefore, it is correct to say that impact of globalization on women is not clear as per the information
provided in the passage.

11.(b)
Assumption 1 is incorrect. This assumption might be true in general, but is not based on the information
given in the passage and is beyond the scope of the passage. The passage nowhere discusses cyber cafes
or their impact on minimizing the digital divide. Hence, this assumption is not correct.
Assumption 2 is correct. The lines “It is suggested that the digital divide may cause technological,
immaterial, social, and educational types of inequality” and “Digital media usage is a strong predictor of
political participation which implies that a digital divide may result in political alienation of those who
lack digital media access” suggest that since digital divide causes technological inequality and results in
political alienation, therefore, bridging it (the digital divide) will lead to technological and political
empowerment.
3 www.visionias.in ©Vision IAS
UPSCMATERIAL.ONLINE

.
12.(d)
Statement 1 is incorrect. This is not a correct assumption because it is beyond the scope of the passage.
The passage nowhere discusses the food security aspect of the future.
Statement 2 is incorrect. This statement might seem to be a correct assumption. However, the lines
“Invasive species may be one of the worst environmental problems facing the conservation of natural
areas, because of their role in changing ecosystem function” suggest that invasive species change the
ecosystem functioning and therefore are one of the worst environmental problems. However, this does not
imply/ guarantee that natural areas can definitely be conserved by keeping a check on the invasive
species. As the lines of the passage clearly suggest invasive species are probably “one of the worst
environmental problems”, it implies that there are other causative factors degrading our environment,
which could also be addressed in order to conserve natural areas.

13.(c)
Statement 1 is correct. The passage mentions, "But technology and its addiction among children are a
hindrance in attaining high EQ level and this leads to many of them not being able to manage their
emotions well”. This line confirms statement 1, that technology acts as a hindrance to the development of
EQ in children.
Statement 2 is correct. The passage mentions, “Relating to others and relationship management is an
important element that nurtures a high EQ in an individual. But technology and its addiction among
children are a hindrance in attaining high EQ level and this leads to many of them not being able to
manage their emotions well". This line also is in line with the given option statement that human relations
have a positive effect on EQ as compared to technology. So, this is the correct assumption.

14.(d)
Prime numbers: A prime number is a positive integer which has no factors other than 1 and itself. The
first few prime numbers are 2, 3, 5, 7, 11, 13, 17 …
Therefore, 2 is the only even prime number.
Now, 46 = 23 × 2; 91 = 13 × 7; 143 = 13 × 11; 153 = 17 × 9
9 is not a prime number.
Hence, option (d) is correct.

15.(a)
Let the length of AB be 100 metres.
Ajay and Vijay are running towards each other at the same speed from points A and B respectively. After
running for some time, Ajay sprains his ankle at point C and walks at half his earlier speed.
Suppose by the time Ajay reaches C, Vijay reaches E and let AC = X.
Thus BE must also be equal to X, as speed of Ajay and Vijay was the same during that interval.
Let, CD = Y
Thus, ED = 2Y as the speed of Vijay from E to D is twice that of Ajay's from C to D.
Hence, Total distance, AB = AC + CD + DE + BE = X + Y + 2Y + X = 2X + 3Y = 100 ……… (i) and
Now, AD = (4/10) AB = (4/10) × 100 = 40 m
AD = AC + CD = X + Y = 40 …………… (ii)
Equation (i) – 2 × equation (ii), we get:
So, X = 20 and Y = 20.
Ratio AC/AD = X:(X + Y) = 20:(20 + 20) = 20:40 = 1:2
Hence, option (a) is correct.

16.(b)
Volume of oil transported by rail = 12 × (9/100) = 1.08 million tonnes
Cost incurred = 30 × (12/100) = 3.6 .
Hence, required answer = 3.6/1.08 = Rs. 3.33 per ton

4 www.visionias.in ©Vision IAS


UPSCMATERIAL.ONLINE

.
17.(a)
Cheapest mode of transportation will be the one which will have the highest transport volume and
comparatively lower cost. We can figure this out from the graphs. But let‟s calculate for the sake of solid
proof.
For Road, Cost = 6/22
For Rail, Cost = 12/9
For Pipeline, = 65/49
For Ship, Cost = 10/9
Lowest per ton cost is for road.

18.(c)
P = 10/9
Q = 7/11
R = 6/22
We can see that P > Q > R.
Hence option (c) is correct.

19.(b)
Ram and Shyam, who are standing 100 m away from each other, are throwing a Frisbee towards each
other in a straight trajectory. The frisbee travels in the air until Ram and Shyam meet.
Let‟s assume that the speeds of Ram and Shyam are a m/s and b m/s respectively.
So, they will meet after [100/(a + b)] seconds.
The frisbee travels throughout this time. The frisbee travelled a total distance of 12 km. They both throw
the frisbee at a speed of 20 m/s.
So, Distance travelled by Frisbee = Time × Speed = [100/(a + b)] × 20 = 12000
or a + b = 1/6 m/s = 10 m/minute
Speeds of Ram and Shyam are in the ratio of 2:3 respectively.
So, a:b = 2:3
Thus, a = 4 m/min and b = 6 m/min
Hence, speed of Shyam is 6 m/min.
Hence, option (b) is correct.

20.(c)
Let a = 10m + n
So, b = 10n + m
It is given that, a2 – b2 = k2
So we have, (10m + n)2 – (10n + m)2 = k2
or 100m2 + n2 + 20mn – (100n2 + m2 + 20mn) = k2
or 99m2 – 99n2 = k2
or 99(m + n)(m – n) = k2
or 11 × 9 × (m + n)(m – n) = k2
9 is already a perfect square, so k2 can be of the form 99×11.
So, k = 33 and (m + n)(m - n) = 11
So, m = 6 and n = 5
a = 10m + n = 10×6 + 5 = 65
So, required numbers are 65 and 56.
Hence, a + b + k = 65 + 56 + 33 = 154
Hence, option (c) is correct.

5 www.visionias.in ©Vision IAS


UPSCMATERIAL.ONLINE

.
21.(c)
Statement 1 is correct. The lines “Crop pollination is commonly cited as an example of an endangered
ecosystem service as detailed studies of the crop pollination systems are incomplete or out of date”, show
that due to incomplete or out of date studies of crop pollination systems this ecosystem service is
endangered. Therefore, the inference given in the statement is correct.
Statement 2 is correct. The passage mentions “Animal pollination is important to the sexual
reproduction of many crops and the majority of wild plants, which can also be important for providing
calories and micronutrients for humans. Furthermore, the decline of pollinating species can lead to a
parallel decline of plant species”. These lines show that pollination of plants has a direct bearing on
human nutrition in terms of the calories and micronutrients available for human beings. If the pollinator
species decline, it will also lead to a reduction in the number of plant species, which in turn provide
nutrition to humans. Hence, this inference is correct.

22.(b)
Statement 1 is incorrect. The passage mentions, “…night time and overcast days can interrupt the
supply”. This indicates that even tropical countries would need to develop storage of solar energy.
Therefore, to infer that they are less dependent on storage will not be correct.
Statement 2 is correct. The passage mentions “As the global capacity for solar power continues to rise,
nations like Japan and other global leaders in solar energy technology are focusing on developing
adequate energy storage to deal with this issue”. This indicates that global leaders in solar energy
technology have started investing in developing adequate energy storage. Therefore, we may infer that, in
the future, the cost of storing solar power would probably decrease.

23.(b)
Option (a) is incorrect. This option is beyond the scope of the passage because there is no discussion on
the supply chain of agricultural produce. So, it is not the central idea.
Option (b) is correct. The lines “Farm automation technology addresses major issues like a rising global
population, farm labour shortages, and changing consumer preferences. The benefits of automating
traditional farming processes are monumental by tackling issues from consumer preferences, labour
shortages, and the environmental footprint of farming”, show that technology will help address the social
(rising global population), economic (farm labour shortages) and environmental issues. So, this captures
the essence of the passage.
Option (c) is incorrect. This statement, in general, would be correct but the passage does not mention
India. So, to extrapolate the benefits of agricultural technology to India without any such clear indication
in the passage would be incorrect.
Option (d) is incorrect. This statement seems to be correct as it would increase the input cost due to the
application of technology. However, this consequence of increased cost due to technology is not covered
in the passage. So, this is not the correct central idea.

24.(a)
Answer can be verified by assuming the values x = 1, y = 3, z = 2.
Let us verify the options:
(a) Since, y is odd and (x – z) = -1, y(x − z)2 is also odd.
Hence option (a) is not true.
(b) Since y and (x − z) both are odd, y2(x − z) is odd.
So, option (b) is true.
(c) Since y and (x − z) both are odd, y(x − z) is odd.
So, option (c) is true.
(d) Since z and (x − y) both are even, z(x − y)2 is even.
So, option (d) is true.
Hence, option (a) is correct.
6 www.visionias.in ©Vision IAS
UPSCMATERIAL.ONLINE

.
25.(d)
Net summation from 1 to 30 = 465. So we are required to add some more numbers.
Summation from 1 to 34 = 595.
Hence, the missed out number = 595 − 575 = 20.
Hence, option (d) is correct.

26.(b)
A number N gives a remainder of 7 when divided by D and a remainder of 20 when divided by 2D.
N = DQ + 7 (where Q = Quotient, and D = Divisor) ………….(1)
N = 2D × K + 20 (where K = Quotient and 2D = Divisor) ……………… (2)
We can say that when N = 2D × K + 20 is divided by D, remainder obtained = 7.
(2DK + 20)/D = (2DK/D) + (20/D)
Remainder obtained from 2DK/D = 0.
Hence, remainder obtained from 20/D = 7
When 20 is divided by 13, then remainder will be 7.
So, D = 13
Now, 2N = 2 (DQ + 7) = 4DQ + 14.
Remainder obtained from 2N/D = Remainder obtained from (4DQ + 14)/D = 14/D = 14/13 = 1.
Hence, the remainder obtained will be 1.
Hence, option (b) is correct.

27.(b)
If there are 22 students in the remedial class, then there will be at least four students with n marks each.
Thus, the minimum number of students who should be sent to the remedial class is 22.
Hence, option (b) is correct.

28.(c)
X and Y are multiples of 7 with X>Y>0.
Let Y = 14 and X = 21
Y = 2×7 and X = 3×7
We will check each option one by one.
Option (a):
X3 – Y3 = 213 – 143 = (7×3)3 – (7×2)3 = 73[33 – 23]
It is divisible by 49. Hence, it is true.
Option (b):
HCF of X and Y = 7. Hence, it is true.
Option (d):
X2 – Y2 = 212 – 142 = (7×3)2 – (7×2)2 = 72[32 – 22]
It is divisible by 49. Hence, it is true.
Option (c):
LCM of X and Y = 7×2×3 = 42
LCM of X and Y is a multiple of 7, but not necessarily a multiple of 49.
Hence, it is not necessarily true.
Hence, option (c) is correct.

29.(b)
Given that a six-digit number N is formed using the digits 0, 3, 6 and 9 only, and each of the digits is used
at least once.
The number is divisible by 18. Therefore, the number has to be divisible by both 2 and 9.
Sum of the digits of the number should be divisible by 9, and the unit‟s digit of the number should be
either 0 or 6.
Smallest possible such number is 300096.
Therefore, the digit at the ten‟s place is 9.
Hence, option (b) is correct.

7 www.visionias.in ©Vision IAS


UPSCMATERIAL.ONLINE

.
30.(c)
Let the numerator of the fraction be x and the denominator be y.
Hence, the original fraction = x/y
New numerator = x + a% of x = x(100 + a)/100
New denominator = y + b% of y = y(100 + b)/100
Hence, new fraction = {x(100 + a)/100}/ {y(100 + b)/100} = (x/y){ (100 + a)/ (100 + b)}
Ratio of original fraction to new fraction = (x/y) / [(x/y){ (100 + a)/ (100 + b)}] = (100 + b)/ (100 + a)
Thus, the required ratio = (100 + b)/ (100 + a)
Hence, option (c) is the correct answer.

31.(b)
Statement 1 is correct. The passage mentions, “With his unbounded greed, man has caused irreversible
damage to the environment. Today, climate change is a global phenomenon.” Here the author highlights
the issue of pollution which is damaging the environment and the reason behind climate change.
Statement 2 is correct. The passage mentions, “Promotion of the mother tongue will revolutionize the
educational landscape by making it more inclusive and equitable.” Here equitable education implies non-
discriminatory education.
Statement 3 is incorrect. The passage mentions, “Rural India cannot be left behind in this mission.
Improving rural infrastructure across the country must be fast-tracked and integrated into the larger
development narrative.” It highlights the issue of rural infrastructure, however, it does not explicitly
highlight the need for irrigation.
Statement 4 is correct. The passage mentions, “Universal and affordable access to quality education and
healthcare must be the focus of our attention.” It implies universal and affordable access to healthcare.
Statement 5 is correct. The passage mentions, “Today, climate change is a global phenomenon.
Conservation is our only hope for the survival of the planet.” It highlights the issue of the survival of the
planet. If the survival of the planet is in danger, then it is an obvious threat to the survival of human
beings.
Statement 6 is correct. The passage mentions, “With his unbounded greed, man has caused irreversible
damage to the environment.” It highlights the issue of human greed which shows his avariciousness
nature.

32.(b)
Option (a) is incorrect. The passage highlights the issue of loneliness in the cities, but it nowhere
prohibits living in cities.
Option (b) is correct. The passage lists out issues of loneliness like people rarely talking to their
neighbours; children do not play cricket; friends may not play together; and friends or relatives are not
generally welcomed. We spend time on online studies or social media. Here, the author criticizes this life
of cities and wants to convey the message of becoming a more socialized person. So, this option best
reflects the suggestion conveyed by the passage.
Option (c) is incorrect. The passage mentions, “…even if, paradoxically, the surfeit of lonely pursuits
has reduced the quality of solitude.” Based on the passage, we cannot say that people are “Searching for
loneliness” – it may be imposed based on the urban cultural setting. Also, this statement can be an
inference but not a suggestion.
Option (d) is incorrect. The passage mentions the problem of loneliness in the cities, so this statement is
correct as the cities have become a ground for loneliness. But this is the problem cited by the author.
While the demand of the question is the suggestion that author wants to convey.

33.(c)
Option (a) is incorrect. The passage talks about „rural‟ being always present in Hindi films, in the past. It
nowhere says that now rural characters are not shown at all.
Option (b) is incorrect. It is true that films affect society and vice versa. But, the existence of a split
between rural and urban being a result of Hindi films ALONE is not conclusively proven in this passage.
Option (c) is correct as the passage does talk about portrayal of „the rural‟ in Hindi films as good, and „the
urban‟ as something evil. This is clear from the 3 rd sentence of the passage.
Option (d) is incorrect due to option (b) being incorrect.
8 www.visionias.in ©Vision IAS
UPSCMATERIAL.ONLINE

.
34.(a)
π (r12 + r22) = 153 π
Or (r12 + r22) = 153
Or (r1 + r2)2 – 2r1r2 = 153
Now, r1 + r2 = 15
So, r1 r2 = 36
On solving, we get:
r1 = 12
r2 = 3
Required Ratio = 4

35.(c)
Let the cost of a caramel pop, a toffee bites and a mango pop be x, y and z respectively.
Using S1:
3x + 5y + 7z = 68 …………….(i)
Using S2:
2x + 3y + 4z = 41 ………….(ii)
We cannot determine the answer from either of the two statements alone.
Using both the statements:
Multiplying equation (ii) by 2 and subtracting it by equation (i), we get:
(4x + 6y + 8z) - (3x + 5y + 7z) = 2 × 41 – 68 = 14
Or x + y + z = 14
So, we can answer the question using both the Statements together, but not by using any of the two
statements individually.
Hence, option (c) is correct.

36.(a)
Percentage changes (rise/fall) in the production of Company Y in comparison to the previous year, for
different years are:
(35 - 25)
For 2007 = x 100 % = 40%.
25
(35 - 35)
For 2008 = x 100 % = 0%.
35
(40 - 35)
For 2009 = x 100 % = 14.29%.
35
(50 - 40)
For 2010 = x 100 % = 25%.
40
Hence, the maximum percentage rise/fall in the production of Company Y was in the year 2007.

37.(c)
Average production of Company X in the period 2008-2010
1 115
= x (25 + 50 + 40) = lakh tonnes.
3 3
Average production of Company Y in the period 2008-2010
1 125
= x (35 + 40 + 50) = lakh tonnes.
3 3

Required ratio = 23:25

9 www.visionias.in ©Vision IAS


UPSCMATERIAL.ONLINE

.
38.(d)
Since value of a lies between – 6 and – 3, so value of a2 must lie between 36 and 9.
And value of b lies between – 3 and 3, so value of b2 must lie between 9 and 0.
To get the upper value of the required range, we will take maximum value of a and minimum value of b:
a2 – b2 = 36 – 0 = 36
To get the lower value of the required range, we will take minimum value of a and maximum value of b:
a2 – b2 = 9 – 9 = 0
∴ Value of (a2 – b2) will lie between 0 and 36.
Hence, option (d) is correct.

39.(b)
Let‟s say that Ramesh bought 100 articles and marked price of each article is Rs. 10.
So, total marked price = 100 × 10 = Rs. 1000
But Ramesh got it for 10% less. So, the amount paid by Ramesh = 0.9 × 1000 = Rs. 900
Now, Ramesh wants to earn a profit of 10%, so he must get 900 ×1.1 = Rs. 990 after selling the articles.
Now, he sold 50% of the articles at a price 10% above the marked price.
Thus, he sold first 50 articles for Rs. 11 each. So amount earned by him = 50 × 11 = Rs. 550
After this he sold the next 25% articles at a price which was 20% lower than the marked price.
Thus, he sold the next 25 articles at Rs. 8 each. So the amount earned by him by selling these articles = 25
× 8 = Rs. 200
So, total amount earned by him by selling 75articles = 550 + 200 = Rs. 750
To earn a profit of 10% in the entire transaction, he needs to sell the remaining 25 articles at 990 – 750 =
Rs. 240.
So, selling price of each of the remaining articles = 240/25 = Rs. 9.6
Marked price is less than the selling price by 10 – 9.6 = Rs. 0.4
Required percentage = (0.4/10) × 100 = 4%
Thus, remaining 25% articles must be sold at 4% below the marked price.
Hence, option (b) is correct.

40.(b)
Let the cost price of 1 kg of rice be C.
For Rani he sells at Rs. C but only sells 0.9 kg instead of 1 kg.
For Vani he sells at Rs. 1.2C but sells 1.1 kg instead of 1 kg.
Overall, he sold 2 kg rice and the cost price for it is Rs. 2C.
His net revenue after both the transactions = C + 1.2C = Rs. 2.2C
Profit = 2.2C – 2C = Rs. 0.2C
Profit percentage = (0.2C /2C)× 100 = 10% profit
Hence, option (b) is correct.

41.(d)
Statement 1 is incorrect. Refer to the lines “Orphan crops are nutritious local food crops that could play
a crucial role in combating hunger. These crops are not traded internationally but have adapted
themselves to grow in harsh weather conditions. Africa‟s arid environment is not suited for crops like rice
and maize. Orphan crops like finger millets, little millet, African yam bean, jojoba and jatropha are
commonly found on the continent.” This shows that orphan crops are nutritious and help in combating
hunger, and they are not traded internationally. However, it does not mean that Africa is not dependent on
any other country. The passage just lays down a few good aspects regarding resistance crops found in
Africa. We cannot draw wide assumptions based on such a narrow premise. Thus, it is an incorrect
assumption.
Statement 2 is incorrect. The lines “Countries like India and Pakistan are also likely to be affected by
the prolonged heatwaves, the United Nations Environment Programme (UNEP) estimated. The UN body
is responsible for coordinating responses to environmental issues”, show that UNEP is responsible for
environment-related issues. But to say that it should promote the trade of orphan crops from Africa to
other hunger-struck countries would not be correct because the UNEP's limited role in the passage is not
about promoting trade. The focus of the passage is more on adoption of drought-resistant genes, rather
than on the trade of such crops. So, this option is not correct as an assumption.
10 www.visionias.in ©Vision IAS
UPSCMATERIAL.ONLINE

.
42.(b)
Statement 1 is incorrect. The statement mentions about completely controlling carbon emissions by 2070
which is an extreme scenario. Net-zero emitter means that there would not be any addition of carbon
emissions to the environment. The passage only talks about goals, but is rather shacky regarding their
achievement. So, to say that with help of renewable energy carbon emissions can be completely controlled
is saying too much too soon. Moreover, it‟s more in the nature of inference than a suggestion. Thus, it is
an incorrect suggestion.
Statement 2 is correct. The lines “But unless people shift their attitude and behaviour in tune with the
policy initiatives, change will not be able to retain its momentum”, reflect that people at individual levels
must change their behaviour and attitude (in line with the policy initiatives) towards climate change. The
option however mentions about government involving citizens in policy formulation and execution, which
may be a valid suggestion in line with the passage. So, this suggestion is correct.

43.(d)
Option (a) is incorrect. This option is beyond the scope of the passage because there is no information
about other countries' contributions to climate change through their commitments. Although, the given
statement is generally correct, to say that climate change cannot be controlled only with India‟s
commitments is not the crucial message of the given passage.
Option (b) is incorrect. This option is also a general option which seems to be correct, but it is not based
on the information given in the passage. The passage is focused on India, its commitment and the need for
policy and people‟s attitudes. Talking about global citizens would not be correct as it is not a part of the
passage. Therefore, this is not the correct crucial message of the passage.
Option (c) is incorrect. The given option is beyond the scope of the passage because there is no mention
of any kind of support from specialised global agencies in the formulation of Indian policy for climate
change. So, this is also not the crucial message.
Option (d) is correct. This option captures the real essence of the passage. The lines “Actions that are
driven top-down, via government policy, no doubt set the stage for change. But unless people shift their
attitude and behaviour in tune with the policy initiatives, change will not be able to retain its
momentum”. These lines reflect that only policy initiatives are not enough to maintain the momentum.
The people should also change their attitude and behaviour towards climate change following the policy
directives. Therefore, the message in this option is the crucial message of the passage.

44.(d)
There are two parts of the quadrilateral: the first one is a right-angled triangle with sides 6, 8 and 10; and
another part is a isosceles triangle with the diameter of the circle as the base and radius as the height.
Area of right-angled triangle = (1/2) × 8 × 6 = 24 cm2
Area of isosceles triangle = (1/2) × 10 × 5 = 25 cm2
So, Maximum possible area of the quadrilateral = 24 + 25 = 49 cm2

Hence, option (d) is correct.

11 www.visionias.in ©Vision IAS


UPSCMATERIAL.ONLINE

.
45.(c)
Let the total number of angles of the polygon be n.
Hence, the sum of the interior angles = 180 × (n - 2)
We know that the number of angles which are 90 degrees is 31.
Hence, the number of angles which are 270 degrees is (n – 31).
Hence, the sum of the interior angles = 90 × 31 + 270 × (n - 31) = 180(n - 2).
Or, 90 × 31 + 270n – 270 × 31 = 180n - 360
Or, 90n = 180 × 31 - 360
Or, n = 2 × 31 - 4 = 58
Hence, the number of angles which are 270 degrees = n – 31 = 58 - 31 = 27
Hence, option (c) is correct.

46.(a)
We know that 30% of the students did not face problems in Social Sciences, 25% of the students did not
face problems in Sciences, 20% students did not face problems in Computers and 10% students did not
face problems in Mathematics.
If each of these students were separate from each other, we would have 30 + 25 + 20 + 10 = 85% people
who did not face a problem in at least one of the four subjects.
Naturally, the remaining 15% would be the students who faced problems in all the four subjects.
Thus, minimum possible percent of the students who faced problems in all the four subjects is 15%.
Hence, option (a) is correct.

47.(a)
There are 4 letters in the word HOME, in which there are two vowels and 2 consonants.
P(first letter is not vowel) = 2/4
P(second letter is not vowel) = 1/3
So, probability that none of letters would be vowels = (2/4) × (1/3) = 1/6
Hence, option (a) is correct.

48.(d)
Let the total population of State R be x million.
Then, population of State R above poverty line
= [(100 - 24)% of x] million
76
= x x million
100
And so, male population of State R above poverty line
2 76
= x xx million
5 100
But, it is given that male population of State R above poverty line = 1.9 million.
2 76 5 x 100 x 1.9
x x x = 1.9 x= = 6.25.
5 100 76 x 2
Total population of State R = 6.25 million.

49.(b)
Total population of State S = 7 million.
Population above poverty line
= [(100 - 19)% of 7] million
= (81% of 7) million
= 5.67 million.
And so, the number of females above poverty line in State S
3
= x 5.67 million
7
= 2.43 million.

12 www.visionias.in ©Vision IAS


UPSCMATERIAL.ONLINE

.
50.(b)
P is a prime number > 71, hence P is an odd number.
(P − 1) and (P + 1) are two consecutive even natural numbers, hence, one of these two numbers will be
divisible by 4 and other by 2.
Also P will be of the format 6K ± 1, hence, one of (P − 1) or (P + 1) = 6K [if 6K + 1, then P − 1 = 6K and
if 6K−1, then P + 1 = 6K].
Since we have already considered these two numbers to be even numbers, hence (P − 1)(P + 1) will be
divisible by 24 (= 2 × 4 × 3)……….. (1)
Now, as P is of the format 6K ± 1, so one of (P + 2) or (P - 2) will be a multiple of 3 ……….(2)
Also, since P is an odd prime number, so unit digit of P = 1 or 3 or 7 or 9.
If unit digit = 3 or 7, then one of (P − 2) or (P + 2) will be a multiple of 5. Or
If unit digit = 1 or 9, then one of (P − 1) or (P + 1) will be a multiple of 5
……………………………….(3)
Combining (1), (2) and (3), N is a multiple of 24 × 3 × 5 = 360.
Hence, N is definitely divisible by 120 and 360. It may or may not be divisible by 240.
Hence, option (b) is correct.

51.(c)
Statement 1 is correct. The passage mentions “Electronic goods and components are the second largest
item, after oil, in India‟s import bill”. So, it is correct to assume that the Indians consume a lot of
electronic goods.
Statement 2 is correct. The passage has made use of the China example to make this suggestion.

52.(c)
Option (a) is incorrect. The passage merely mentions “India which will soon have twice the number of
Internet users as in the U.S. is a large market for all kinds of new technologies”. There is no information
about any kind of electronics or technological services trade between the USA and India. Hence, this
option is beyond the scope of the passage. So, this is not the crux of the passage.
Option (b) is incorrect. This option is also not correct because there is no information about issues of
human resources which hampers the functioning of domestic industry. Hence, it is not the crux of the
passage.
Option (c) is correct. The lines “India which will soon have twice the number of Internet users as in the
U.S. is a large market for all kinds of new technologies”, “For instance, the country is operating far
below its potential in electronic manufacturing” and “High-value electronic components needed in the
manufacture of, say, mobile phones are technology and design-intensive. Big multinational companies
control these technologies and corner the bulk of the revenues”, show that despite a large user base,
India is not able to exploit the potential of the electronic industry which is mostly controlled by big
multinational companies. Therefore, this is the crux of the passage.
Option (d) is incorrect. The given option is beyond the scope of the passage because there is no
discussion on the aspect of manufacturing bases of big multinational companies. Hence, this is not the
crux of the passage.

53.(c)
Statement 1 is correct. Refer to the lines "Three factors that largely determine the efficacy of any system
of governance are the quality of leadership, the characteristics of the governed, and the nature of the
structures and processes employed to exercise authority and meet human needs”. The characteristics of
the governed is reflecting the citizens of the country. Thus, the given assumption in the statement is
correct because governance depends on three factors: the quality of leadership, the characteristics of the
governed, and the nature of the structures and processes employed to exercise authority and meet human
needs. So, citizens are also responsible for efficient outcomes of governance.
Statement 2 is correct. The passage mentions, “The capacity of any institution to effect and manage
change, and to respond creatively to challenges that lie before it, increases the development of several
critical skills. These include the ability to maintain a clear perception of social reality and the forces
operating in it; to properly assess the resources of the community; and to implement decisions with an
openness and flexibility that avoid all traces of dictatorial behaviour, among others". So, the capacity of
the institution increases if the implementation of decisions is done with openness and flexibility.
Therefore, it would be correct to say that capacity will reduce if the implementation is done in a rigid and
closed manner. So, this is the correct assumption.

13 www.visionias.in ©Vision IAS


UPSCMATERIAL.ONLINE

.
54.(d)
Total number of balls = 90
The numbers that give the same units digit on being squared are 0, 1, 5, 6. Hence the chance of getting
such a number in units digit is 4/10 = 2/5.
After the first ball is chosen, there would be 9 – 1 = 8 balls remaining with that unit digit (out of 89).
Hence, the probability that the second number has the same units digit = 8/89
Required probability = (2/5) × (8/89) = 16/445
Hence, option (d) is correct.

55.(b)
Let the efficiency of each person be 1 unit of work per day.
So, number of units of work to be done in total = 100 units
Work done on the first day = 1 unit
Work done on the second day = 2 units
Work done on the third day = 3 units and so on
Let the work be completed in „n‟ days
Total Work = 1 + 2 + 3 +…+ n = n(n+1)/2 units
So, n(n+1)/2 = 100
or n(n+1) = 200
or 14×15 ≥ 200
So, minimum numerical value of n is 14 days.
So, the work gets completed on the 14th day.
Hence, option (b) is correct.

56.(b)
Given: x + y = 36
Statement 1: Taking x = 37 and y = – 1
x + y = 37 + (– 1) = 36
Hence, statement 1 is not true because one of x or y can be negative.
Statement 2: Taking x = 35 and y = 1
x + y = 35 + 1 = 36
Hence, statement 2 is not true because y can be positive even if x is positive.
Statement 3: It must be true because y has to be positive if x is negative. Only then their summation can
be positive.
For example, taking x = – 12 and y = 48
x + y = – 12 + 48 = 36
Hence, option (b) is correct.

57.(a)
Let the first term of the AP be „a‟ and the common difference be „d‟.
The sixth term of the series will be a + 5d.
Given that, „d‟ should be a factor of „a + 5d‟. That is, a + 5d is divisible by d.
So, a should be divisible by d.
So the required cases are
d = 1, a = 1, 2, 3.......20 → Number of cases = 20
d = 2, a = 2, 4, 6.......14 → Number of cases = 7
d = 3, a = 3, 6, 9 → Number of cases = 3
d = 4, a = 4 → Number of case = 1
So, the required number of AP‟s = 20 + 7 + 3 + 1 = 31.
Hence, option (a) is correct.

14 www.visionias.in ©Vision IAS


UPSCMATERIAL.ONLINE

.
58.(d)
Ratio of incomes of Anil and Ajit is 5:7 and the ratio of their expenditures is 3:4.
Let the incomes be 5k and 7k and expenditures be 3t and 4t.
Savings = Income - Expenditure
Ratio of savings = (5k-3t)/(7k-4t).
Let us assume that (5k-3t)/(7k-4t) = 4/5 (Option a).
25k - 15t = 28k -16t
Or t = 3k.
Now, substituting this relationship in the equation (5k-3t)/(7k-4t), we get -4k/-5k = 4/5. Though the
overall ratio is 4/5, the numerator is -4k. Which means that, 3t > 5k in this case. Same is the case with
denominator. However, we know that expenditure cannot be greater than income and therefore, we can
eliminate this case. Checking similarly, we can eliminate options (b) and (c).
Option (d):
(5k - 3t)/(7k - 4t) = 12/17
Or 85k - 51t = 84k - 48t
Or k = 3t
Substituting this relationship in (5k - 3t)/(7k - 4t), we get the ratio as 12t/17t.
Both the numerator and denominator are positive and hence, option (d) is a possible solution.
Hence, option (d) is correct.

59.(b)
Let the number of paintings in which both kind of colours have been used be x. With the help of Venn
diagram, the given information can be represented as follows:

So, (16 – x) + x + (19 – x) = 29


or 35 – x = 29
or x = 35 – 29 = 6
Hence, number of paintings in which both type of colours have been used = 6
Hence, option (b) is correct.

60.(b)
Let the two numbers be „a‟ and „b‟ respectively.
There are 2 possibilities:
Case 1: ab - a - b = 72
or ab - a - b + 1 = 73
or (a - 1)(b - 1) = 1 × 73 = (-1) × (-73)
a = 2, b = 74 and a = 0, b = - 72 are the solutions.
Case 2: a + b - ab = 72
or ab - a - b + 1 = -71
or (a - 1)(b - 1) = 1 × (-71) = (-1) × 71
a = 2, b = -70 and a = 0 and b = 72 are the solutions.
Thus, there are 4 pairs of such integers.
Hence, option (b) is correct.
15 www.visionias.in ©Vision IAS
UPSCMATERIAL.ONLINE

.
61.(a)
Option (a) is correct. The passage mentions, “The ignorance of class differences that are necessitated in
democracy‟s pursuit of equality makes it vulnerable to the class conflict” and “The responsibility to
uphold the law, typically maintained in the form of a constitution, by both the government and the
individual is essential in a republic for the preservation of societal order”. These lines show that
democracy has some limitations, but a constitution which helps the government and its citizens to uphold
the law eventually helps in maintaining social order. So, this is the crux of the passage.
Option (b) is incorrect. This option seems to be correct due to the following lines “The ignorance of
class differences that are necessitated in democracy‟s pursuit of equality makes it vulnerable to class
conflict. By institutionalizing class disparities into a balanced representative structure, republicanism
attempts to preserve order between distinctive subdivisions of society. Also, the pursuit of complete
„negative liberty‟ in pure democracies is susceptible to the anarchic disorder, as it disregards law”.
However, there is no comparison of democracy as a governance model with any other model in the
passage to conclude whether democracy is the best model or not. Hence, it is not the crux of the passage.
Option (c) is incorrect. This option is beyond the scope of the passage because there is no information in
the passage that only the governments following a constitution guarantees the preservation of social order.
Therefore, this cannot be the crux of the passage.
Option (d) is incorrect. The given option is contradictory to the lines of the passage “The ignorance of
class differences that are necessitated in democracy‟s pursuit of equality makes it vulnerable to the class
conflict” where the pursuit of equality is the reason for class conflicts. Hence, this is not the crux of the
passage.

62.(a)
Option (a) is correct. The passage mentions “In India, nearly 60% of the household waste is wet organic
waste, with low calorific value. This makes options such as waste-to-energy incinerators inefficient”. It
reflects that incineration in India is not the best way to dispose of organic waste. So, this is the crux of the
passage.
Option (b) is incorrect. This option can be an inference but not the crux as it does not tell the main theme
(Incineration of waste in India) of the passage. Reducing consumption can be a solution in general but that
is not suggested in the passage. Hence, it is not the crux of the passage.
Option (c) is incorrect. There is no discussion on the location of incinerators being done in the passage.
So, this option is beyond the scope of the passage. Therefore, this cannot be the crux of the passage.
Option (d) is incorrect. The given option could be a fair suggestion. However, the passage does not
mention the development of customized incinerators. The crux is about the given information and not
about possible implications from the given passage. Hence, this is not the crux of the passage.

63.(a)
Option (a) is correct. This is the correct option because it builds on the core idea of the passage. The
lines “Most NCDs primarily result from unhealthy lifestyles including the consumption of too much or
unhealthy food, too much alcohol and excessive smoking habits, combined with physical inactivity”, show
that NCDs result from unhealthy lifestyles like physical inactivity and unhealthy food. So, prevention of
NCDs can be done through physical activity and healthy food choices.
Option (b) is incorrect. There is no lesson for India in the given passage because there is no data to
reflect India‟s situation. Also, there is no discussion about India in the passage.
Option (c) is incorrect. This statement is incorrect. The passage mentions, “Especially in the last century,
most Western countries have experienced significant demographic changes with a continuing increase in
the number of older people who face medical and functional challenges, as well as diseases that are age-
specific but have often originated in people‟s younger years”. It merely shows that some age-specific
diseases can even develop early. So, the option is incorrect because it talks about the development of any
kind of disease at any stage of life.
Option (d) is incorrect. This statement is incorrect. The passage mentions, “More specifically, inactivity
and unhealthy eating habits are associated with weight gain, overweight and obesity are the major
underlying causes for modern diseases such as CHD or type 2 diabetes mellitus.” The option given could
be correct in general, but the passage mentions that weight gain, overweight and obesity are the major
causes of CHD, and diabetes and not unhealthy food, alcohol, or smoking. Moreover, this is just a fact
stated in the passage; it cannot be called a practical implication.

16 www.visionias.in ©Vision IAS


UPSCMATERIAL.ONLINE

.
64.(d)
Using S1 alone:
It does not provide the answer as the price of mixture C is not known.
Using S2 alone:
It does not provide the answer as the price of mixtures A and B are not known.
Using S1 and S2 both:
It can be observed that mixing A and B will get any price between Rs. 32 and Rs. 40. For example, Rs. 33
per litre, or Rs. 34 per litre, or Rs. 35 per litre, etc. Mixing it with C will make the average price of the
mixture as Rs. 35 per litre. But ratio of A : B : C in each of these mixtures will be different.
Thus, S1 and S2 together are NOT sufficient to answer the question.
Hence, option (d) is correct.

65.(b)
Prime numbers x and y are greater than 2, hence they must be odd. Difference of two odd numbers is
even. So, option (a) is definitely false.
The sum of two odd numbers is even. So, option (b) must be true.
Hence, option (b) is correct.

66.(b)
If the product a and b is greater than 0, i.e. ab > 0, then both a and b must have the same sign.
It means either a > 0, b > 0 OR a < 0, b < 0.
Using S1 alone:
a > b3 ⇒ a > b
Now, there are two possibilities:
Either (i) both a and b are positive. In that case, a + b > 0;
(ii) Both a and b are negative.
Assume a = −5 and b = −10.
So, a + b = −15 < 0
Hence, S1 alone is not sufficient.
Using S2 alone:
Given that a > b6
So, b6 is positive and a is greater than it. So, a > 0
It will be possible when both a and b will be positive, because their product is a positive number.
So, a + b > 0.
Hence, question can be answered by using S2 alone.
Hence, option (b) is correct.

67.(c)
21 divides the number n.
So, the smallest divisor of n is either 2 or 3 (as 3 divides 21)
So, the largest divisor of n is either 21×2 = 42 or 21×3 = 63
Hence, n = 84 (21×2 = 42) or n = 189 (21×3=63)
So, number of possible values of n is 2.
Hence, option (c) is correct.

68.(a)
Let the work done together by 10 men and 20 women in a day be W.
10 men and 20 women together complete a project in 48 days.
Total work = 48W
After the first day, work remaining = 47W.
Because of the replacement, let the additional work that can be done be D.
So, 47W = 45(W + D)
Or D = 2W/45
Similarly, number of extra days needed in the second case = 46W/[W + 2D] = 46W/[W + 2 × (2W/45)] =
42.2
So, the number of extra days taken to finish the project will be 43 days (round up to the nearest integer).
Hence, option (a) is correct.

17 www.visionias.in ©Vision IAS


UPSCMATERIAL.ONLINE

.
69.(d)
Cost price of the substance = x
Assuming the number of quantity processed as y, the labour cost as L and the other cost as m.
Cost price of the substance = (Labour cost + Other cost)/Quantity of the substance processed = x
So, (L + m)/y = x
Here, L = 40 hours
or, a + b = x, (Assuming L/y = a and m/y = b)
Since y and m remain the same. Only L is reduced to half (i.e. L becomes 20 from 40).
Then, a/2 + b = a/2 + b/2 + b/2 = [(a + b)/2] + b/2 = x/2 + b/2
From the above equation, it is clear that the new price of the processed substance per unit quantity will be
greater than x/2.
Hence, option (d) is correct.

70.(b)
Let the cost of a pencil, a eraser and a sharpener be a, b and c respectively.
The cost of 4 pencils, 2 erasers and a sharpener together is Rs. 80.
So, 4a + 2b + c = 80 ---(i)
The cost of 1 pencil, 2 erasers and a sharpener together is Rs. 35.
a +2b + c = 35 ---(ii)
Subtracting equation (i) and (ii), we get:
3a = 45
or a = 15
Now, the cost of 2 pencils, 2 erasers and 1 sharpener will be 2a + 2b + c.
Adding “a” to equation (ii), we will get:
a + a + 2b + c = 35 + 15
or 2a + 2b + c = 50
Thus, the combined cost of 2 pencils, 2 erasers and 1 sharpener is Rs. 50.
Hence, option (b) is correct.

71.(a)
Assumption 1 is correct. The passage mentions, “Despite abundant low-skilled labour, our growth
trajectory has mostly skipped manufacturing.” It clearly implies that availability of low-skilled labours
would encourage the growth of manufacturing sector.
Assumption 2 is correct. The passage mentions, “Growing instead on the back of a far smaller, high-
skilled services sector.” It implies that the service sector is thriving on a small workforce.
Assumption 3 is incorrect. The passage mentions, “Growing instead on the back of a far smaller, high-
skilled services sector. Consequently, the bulk of jobs our economy generated even in its peak growth
years was in the largely informal, low value add construction sector.” It implies that the economy post-
1991 is growing on the back of the service sector. But the passage does not mention the peak growth
years, so we cannot assume whether it was the period from 2005-2012 or not.

72.(a)
Statement 1 is correct. The passage mentions, “However, as the World Bank data show, most of those
(40 per cent of the population) who escaped poverty between 2005-2012 moved into what they call the
vulnerable group — one income shock away from falling below the poverty line.” It implies that there is a
group of people who are on the border of poverty line. These people are working in the informal, low-
value adding construction sector. So, any income shock can push them back into poverty.
Statement 2 is incorrect. Although the lines, “Despite abundant low-skilled labour, our growth
trajectory has mostly skipped manufacturing, growing instead on the back of a far smaller, high-skilled
services sector” highlight the growth of services sector, they do not mention how the agriculture sector
has performed in India during the same period. Therefore, no conclusive remarks can be made with regard
to status of the agricultural sector.

18 www.visionias.in ©Vision IAS


UPSCMATERIAL.ONLINE

.
73.(a)
Statement 1 is correct. The lines “What we need to understand, and question is the politics behind
recognizing the domain of intellect as essentially a male prerogative”, show that intellect has been a male
forte which created inequality, and that is why we need to change it to give women equal importance in
the area of intellect. Therefore, this inference is correct.
Statement 2 is correct. The passage mentions “And of course, the mind is superior to the body. In this
regard, the feminist critique of French philosopher René Descartes‟s Cartesian duality is an important
intervention. The mind-body dualism has led to gender inequality for privileging the ‘rational’ mind
over the ‘irrational’ or emotional body.” This establishes that due to the mind-body dualism of Descartes
there has been a division in society which led to the belief of men being superior to women. Thus, this
statement as inference is correct.
Statement 3 is incorrect. This inference is incorrect because the passage does not discuss the social
dimension of gender inequality anywhere. It might be correct in general, but we need to answer within the
context of the given passage. Therefore, this inference is not correct.

74.(b)
Statement 1 is incorrect. The given option is not correct because the statement is extreme.
Statement 2 is correct. The passage mentions “Thus, education is not merely learning of facts but is to
train our mind to think. Education systems must provide opportunities to every individual to learn through
experience and should help to develop critical thinking and problem-solving skills” and “Instead of
focusing on critical thinking, expressing new ideas and debating and writing critically on any issue, our
students are forced to learn through the rote route”. These lines show that to develop critical thinking
(outcome of education) one should resort to experienced-based learning and reduce rote learning.

75.(c)
Let the initial average weight of 150 students be x.
Sum of the weights of 150 students = 150x
Let the new student, who is added, has weight m.
The average weight of the new class becomes x + 0.05
Therefore, [150x + m]/151 = x + 0.05
Or 150x + m = 151x + 7.55
Or m = x + 7.55
Let the weight of the student who is removed be k. Then the new average becomes x + 0.05 - 0.02 = x +
0.03
According to the question,
Or [150x + m - k]/150 = x + 0.03
Or m - k = 150 × 0.03 = 4.5
Or k = m - 4.5 = x + 7.55 - 4.5 = x + 3.05 (Since, m = x + 7.55)
Now sum of the weights = m + k = (x + 7.55) + (x + 3.05) = 2x + 10.6
Sum of the weights of the two students mentioned is 2x + b.
On comparing with the above equation, we get b = 10.6.
Hence, option (c) is correct.

76.(a)
Each of the three varieties contain starch, hops and water in varying proportions
A contains x% starch and y% hops.
So, percentage of water in A = [100 - (x + y])%.
B contains y% starch and y% water.
So, percentage of hops in B = [100 - 2y]%
C contains x% hops and y% water.
So, percentage of starch in C = [100 - (x + y)]%
Concentration of starch in final mixture is 38%, so
(2x/10) + (3y/10) + 5{100 - (x + y)}/10) = 38
Or 3x + 2y = 120 ………………(i)
Also, given concentration of hops in final mixture is 28%, so
(2y/10) + 3(100 - 2y)/10 + (5x/10) = 28
Or -5x + 4y = 20 …………….(ii)
Solving (i) and (ii), we get:
x = 20 and y = 30
∴ x + y = 20 + 30 = 50
Hence, option (a) is correct.
19 www.visionias.in ©Vision IAS
UPSCMATERIAL.ONLINE

.
77.(d)
Here we have to select, i.e. it's a question of combination (and not permutation), as the order of the players
do not matter.
As the team must have at least 5 bowlers, two cases emerge:
Case I: Team has 5 bowlers and 6 batsmen.
Number of ways of choosing such a team = Number of ways of choosing 5 bowlers out of 6 available and
6 batsmen out of 7 available = 6C5 × 7C6 = 6 × 7 = 42.
Case II: Team has 6 bowlers and 5 batsmen.
Number of ways of choosing such a team = Number of ways of choosing 6 bowlers out of 6 available and
5 batsmen out of 7 available = 6C6 × 7C5 = 1 × 21 = 21.
Hence, total possible ways = 42 + 21 = 63.
Hence, option (d) is correct.

78.(a)
Percentage share of small establishments in jobs in 1981 = 20%
So, Number of jobs in small establishments in 1981 = (20/100) × 50 = 10 million
And percentage share of small establishments in jobs in 1987 = 30%
So, Number of jobs in small establishments in 1987 = (30/100) × 60 = 18 million
Required difference = 18 – 10 = 8 million
Hence, option (a) is the correct answer.

79.(a)
Percentage share of medium establishments in jobs in 1996 = 75%
Total number of employees in manufacturing sector in 1996 = 100000
So, Number of employees in medium establishments in 1996 = (75/100) × 100000 = 75000
Now, Number of medium establishments in 1996 = 750
Average number of employed persons per medium establishment unit in that year = 75000/750 = 100
Hence, option (a) is the correct answer.

80.(b)
The first story starts on the 2nd page, i.e. from an even page number.
We have to find the largest number of stories than can begin on an odd numbered page. Each story other
than the first story (remaining 56 stories), starts on a fresh page.
Thus, there could be a maximum of 56 stories starting on an odd-numbered page.
Hence, option (b) is correct.

Copyright © by Vision IAS


All rights are reserved. No part of this document may be reproduced, stored in a retrieval system or transmitted
in any form or by any means, electronic, mechanical, photocopying, recording or otherwise, without prior
permission of Vision IAS.

20 www.visionias.in ©Vision IAS

You might also like